Velocity of end points of rectangular frames

In summary: I can't seem to type it correctly. Let ##ρ## be the line density.Moment of inertia of AB and CD: ##(2b\rho )4b^2/12=2b^3\rho/3##Moment of inertia of BC and AD: ##(c\rho )b^2##Hence the moment of inertia of the... sorry, I can't seem to type it correctly. Let ##ρ## be the line density.Moment of inertia of AB and CD: ##(2b\rho )4b^2/12=2b^3\rho/3##Moment of inertia of BC and
  • #1
Saitama
4,243
93

Homework Statement


(see attachment, ignore those marks done with the pen)


Homework Equations





The Attempt at a Solution


I think this has to do with calculating torques and forces but I don't know about which point to calculate torque about. I know this is a very less attempt towards the problem but I can't really figure out the point about which to calculate torque about. :confused:
 

Attachments

  • rotation.jpg
    rotation.jpg
    47.4 KB · Views: 385
Physics news on Phys.org
  • #2
Hi Pranav-Arora! :smile:
Pranav-Arora said:
I think this has to do with calculating torques and forces but I don't know about which point to calculate torque about. I know this is a very less attempt towards the problem but I can't really figure out the point about which to calculate torque about. :confused:

Can't you simply use conservation of energy?
 
  • #3
Hi tiny-tim!

tiny-tim said:
Can't you simply use conservation of energy?

Initial energy=##2mgb\cos(\theta/2)## where m is the mass of one frame. (U=0 on the smooth horizontal surface)
Final energy=##mv_{cm}^2##

Equating them doesn't give the right answer. :confused:
 
  • #4
Pranav-Arora said:
Final energy=##mv_{cm}^2##

nooo …

isn't it also rotating? :wink:
 
  • #5
If using conservation of energy, one has to demonstrate that the system is conservative, not just take that for granted.
 
  • #6
tiny-tim said:
nooo …

isn't it also rotating? :wink:

Oh yes, I was wondering as to why they gave the value c. :P

The moment of inertia came out to be ##20m/3## (Do I have to calculate the moment of Inertia when the frames lie on the ground? This is what I have done to calculate 20m/3.)

[tex]2mgb\cos(\theta/2)=mv_{cm}^2+I\omega^2[/tex]
Replacing ##\omega## with ##v_{cm}/b##, I don't end up with a nice answer. Is there a mistake in my calculated moment of inertia or in my arithmetic? :confused:

voko said:
If using conservation of energy, one has to demonstrate that the system is conservative, not just take that for granted.

How to do that? I have never encountered this type of question before where you need to show that system is conservative.
 
  • #7
Pranav-Arora said:
Oh yes, I was wondering as to why they gave the value c. :P

The moment of inertia came out to be ##20m/3##

how did you get that? :confused:

(and the moment of inertia, and rotational energy, must be calculated about the centre of mass)
voko said:
If using conservation of energy, one has to demonstrate that the system is conservative, not just take that for granted.

sorry, i disagree …

the question says the system is frictionless (and there's no sharp collision), so it obviously intends conservation of energy to be assumed
 
  • #8
Pranav-Arora said:
How to do that? I have never encountered this type of question before where you need to show that system is conservative.

You can't apply conservation of energy unless you know energy is conserved.

This is done by considering all the forces acting on the system. Either they must be potential, or their work must always be zero.
 
  • #9
tiny-tim said:
how did you get that? :confused:

(and the moment of inertia, and rotational energy, must be calculated about the centre of mass)

Yes, I know it should be calculated about CM but I am not sure about which axis should I do it. Should it be about the axis shown by a dotted line in the question?
 
  • #10
Pranav-Arora said:
… Should it be about the axis shown by a dotted line in the question?

yes :smile:

but i don't see what you think the other choices might be :confused:

the axis must be parallel to the angular velocity (which is clearly parallel to that line), and there's only one line parallel to a particular line through a particular point

(and i assume you're treating each half separately?)
 
  • #11
tiny-tim said:
yes :smile:

but i don't see what you think the other choices might be :confused:

the axis must be parallel to the angular velocity (which is clearly parallel to that line), and there's only one line parallel to a particular line through a particular point

(and i assume you're treating each half separately?)

See attachment for the sketch.

The moment of inertia of AB and CD about the axis is ##4mb^2/12=mb^2/3##
The moment of inertia of BC and AD is ##mb^2##

Hence, the moment of inertia of one rectangular frame is ##13mb^2/12##. Is this correct?
 

Attachments

  • rectangle1.png
    rectangle1.png
    2.1 KB · Views: 369
Last edited:
  • #12
Pranav-Arora said:
The moment of inertia of AB and CD about the axis is ##4mb^2/12=mb^2/12##
The moment of inertia of BC and AD is ##mb^2##

yes :smile:
Hence, the moment of inertia of one rectangular frame is ##13mb^2/12##. Is this correct?

i'd be happier if you used ρ for line density, rather than m …

i'm not sure what your m is supposed to be :confused:

anyway, can you please show your working
 
  • #13
ooh, just occurred to me … ω isn't vc.o.m/b :wink:

(and now I've got to go and watch doctor who on telly, and then I'm going out for the rest of the evening)
 
  • #14
tiny-tim said:
yes :smile:i'd be happier if you used ρ for line density, rather than m …

i'm not sure what your m is supposed to be :confused:

anyway, can you please show your working

Sorry I did a lot of mistakes in my previous post. :redface:

Let ##ρ## be the line density.
Moment of inertia of AB and CD: ##(2b\rho )4b^2/12=2b^3\rho/3##
Moment of inertia of BC and AD: ##(c\rho )b^2##

Hence the moment of inertia of the rectangular frame is ##\displaystyle \frac{2b^3\rho}{3}+\frac{2b^3\rho}{3}+c\rho b^2+c\rho b^2=\frac{4b^3\rho}{3}+2c\rho b^2=\frac{2b^2\rho (2b+3c)}{3}##

where ##\rho=\frac{m}{2(2b+c)}## and m is the mass of one rectangular frame.

timy-tim said:
ooh, just occurred to me … ω isn't vc.o.m/b :wink:

Why? What is it?
 
  • #15
(just got back :biggrin:)
Pranav-Arora said:
Why? What is it?

sorry, i was in a hurry and i got confused :redface:

at the instant the frame hits the ground, one end of the frame is stationary, so ω = vc.o.m/b does work
 
  • #16
Sorry for the late reply, I had a test.

tiny-tim said:
sorry, i was in a hurry and i got confused :redface:

at the instant the frame hits the ground, one end of the frame is stationary, so ω = vc.o.m/b does work

We had
[tex]2mgb\cos(\theta/2)=mv_{cm}^2+I\omega^2[/tex]
Substituting the value of I and replacing ##\omega## with v_(cm)/b
[tex]2mgb\cos(\theta/2)=mv_{cm}^2+\frac{2b^2\rho (2b+3c)}{3}\frac{v_{cm}^2}{b^2}[/tex]
[tex]\Rightarrow 2mgb\cos(\theta/2)=mv_{cm}^2+\frac{m(2b+3c)}{3(2b+c)}v_{cm}^2[/tex]
[tex]\Rightarrow 2\times 10 \times \frac{4}{5}=\frac{3}{2}v_{cm}^2[/tex]

This doesn't give me a nice value for v_(cm). :confused:
 
  • #17
(and v = 2vcm)

hmm, it doesn't match any of the choices :confused:

sorry, i can't see the mistake
 
  • #18
I have uploaded the solution presented in the solution booklet. I couldn't understand it so I had to post the question here. Can you please explain this? :)
 

Attachments

  • Picture1.jpg
    Picture1.jpg
    27.8 KB · Views: 325
  • #19
ah, i see it now …
Pranav-Arora said:
[tex]\Rightarrow 2mgb\cos(\theta/2)=mv_{cm}^2+\frac{m(2b+3c)}{3(2b+c)}v_{cm}^2[/tex]
[tex]\Rightarrow 2\times 10 \times \frac{4}{5}=\frac{3}{2}v_{cm}^2[/tex]

you left out the b (= 3/2) on the LHS :wink:
 
  • #20
tiny-tim said:
ah, i see it now …you left out the b (= 3/2) on the LHS :wink:

Thanks a bunch tiny-tim! :smile:

I should have taken care. :redface:

The answer comes out to be 8 m/s.
 
  • #21
btw, in case you're wondering …

the book takes moment of inertia etc about the endpoint, using the parallel axis theorem

we used the centre of mass

obviously, they came out the same!

you can always use the centre of mass (as we did),

but you can only use another point if it's the instantaneous centre of rotation (as the end is, but only at the instant the frame is flat)
 
  • #22
tiny-tim said:
btw, in case you're wondering …

the book takes moment of inertia etc about the endpoint, using the parallel axis theorem

we used the centre of mass

obviously, they came out the same!

you can always use the centre of mass (as we did),

but you can only use another point if it's the instantaneous centre of rotation (as the end is, but only at the instant the frame is flat)

Thank you once again! :)
 

What is the velocity of end points of rectangular frames?

The velocity of end points of rectangular frames is the rate of change of position of the end points of a rectangular frame over time.

How is the velocity of end points of rectangular frames calculated?

The velocity of end points of rectangular frames can be calculated by taking the derivative of the position function of the frame with respect to time.

What factors can affect the velocity of end points of rectangular frames?

The velocity of end points of rectangular frames can be affected by the size and shape of the frame, the force applied to the frame, and any external forces acting on the frame such as friction or air resistance.

Is there a maximum velocity for end points of rectangular frames?

There is no specific maximum velocity for end points of rectangular frames, as it depends on the specific frame and the forces acting on it. However, there may be practical limits based on the strength and durability of the frame materials.

What is the significance of studying the velocity of end points of rectangular frames?

Studying the velocity of end points of rectangular frames is important in fields such as engineering and physics, as it allows us to analyze and predict the motion of objects that have a rectangular frame. It also helps us understand the dynamics of rectangular frames and their behavior in different situations.

Similar threads

  • Introductory Physics Homework Help
Replies
5
Views
999
  • Introductory Physics Homework Help
Replies
30
Views
2K
  • Introductory Physics Homework Help
Replies
4
Views
264
  • Introductory Physics Homework Help
Replies
17
Views
714
  • Introductory Physics Homework Help
Replies
1
Views
333
  • Introductory Physics Homework Help
Replies
20
Views
950
  • Introductory Physics Homework Help
Replies
25
Views
259
  • Introductory Physics Homework Help
Replies
32
Views
1K
  • Introductory Physics Homework Help
Replies
11
Views
213
  • Introductory Physics Homework Help
Replies
10
Views
1K
Back
Top